Nontrivial Twists of a Vector BundleTwists of rational pointsG-equivariant bundleProjective bundle is projective?Comparing notions of degree of vector bundleCohomologies with line bundle vs. coherent coefficientsWhat is Higgs fieldDefinition of the hyperplane bundle in algebraic geometryTwists of etale cohomologyEtale cohomology of a number fieldCohomology of pushforwards of vector bundles over the projectivization of a vector bundle

Is there a way to make member function NOT callable from constructor?

How to create a consistant feel for character names in a fantasy setting?

Check if two datetimes are between two others

Crop image to path created in TikZ?

What to wear for invited talk in Canada

Patience, young "Padovan"

OA final episode explanation

What is the meaning of "of trouble" in the following sentence?

Why airport relocation isn't done gradually?

Is this homebrew feat, Beast of Burden, balanced?

Correctly defining the return of a procedure

Wild Shape Centaur Into a Giant Elk: do their Charges stack?

My colleague's body is amazing

What do you call something that goes against the spirit of the law, but is legal when interpreting the law to the letter?

Why is making salt water prohibited on Shabbat?

Are objects structures and/or vice versa?

extract characters between two commas?

Shall I use personal or official e-mail account when registering to external websites for work purpose?

Extreme, but not acceptable situation and I can't start the work tomorrow morning

Landlord wants to switch my lease to a "Land contract" to "get back at the city"

Is Fable (1996) connected in any way to the Fable franchise from Lionhead Studios?

aging parents with no investments

Is domain driven design an anti-SQL pattern?

Sort in WP_Query(), not filter? Is it possible?



Nontrivial Twists of a Vector Bundle


Twists of rational pointsG-equivariant bundleProjective bundle is projective?Comparing notions of degree of vector bundleCohomologies with line bundle vs. coherent coefficientsWhat is Higgs fieldDefinition of the hyperplane bundle in algebraic geometryTwists of etale cohomologyEtale cohomology of a number fieldCohomology of pushforwards of vector bundles over the projectivization of a vector bundle













6












$begingroup$


Let $k$ be a number field, and let $X$ be a projective $k$-variety. Let $mathcalV$ be a vector bundle on $X$ that is defined over $k$. A vector bundle $mathcalV'$ on $X$ that is defined over $k$ is said to be a twist of $mathcalV$ if there is some finite extension $ell/k$ such that we have an isomorphism $mathcalV_ell simeq mathcalV'_ell$ of bundles upon basechanging to $ell$.



Question: Is it true that all twists of $mathcalV$ are in fact isomorphic to $mathcalV$ over $k$?



What I know: Twists are parameterized by the Galois cohomology group $H^1(operatornameGal(overlinek/k),operatornameAut(calV_overlinek))$, so all we need to do is determine whether this cohomology group is equal to $0$. It's a consequence of Hilbert Theorem 90 that $H^1(operatornameGal(overlinek/k),operatornameGL_n(overlinek)) = 0$, which might be useful.










share|cite|improve this question









$endgroup$







  • 1




    $begingroup$
    And you really need this full statement? I see how to prove the answer in the affirmative if you assume that $X$ is integral and either you're dealing with line bundles or one of your vector bundles is trivial. In the complete general case this, at first glance, seems kind of tough. You're essentially asking about the Galois cohomology of the algebraic group $Rmapsto Aut(mathcalV_R)$. Are there large constraints on what this algebraic group can be?
    $endgroup$
    – Alex Youcis
    Mar 30 at 6:12











  • $begingroup$
    @AlexYoucis I'm happy to assume $X$ is integral, and I think I know how to handle the cases where $calV$ is a line bundle or is trivial. I'm interested primarily in the case where $calV$ splits completely as a direct sum of line bundles, but I'm not sure whether that constrains $operatornameAut(V)$ sufficiently. Does your argument for the line bundle case generalize to the case of the direct sum of line bundles?
    $endgroup$
    – Ashvin Swaminathan
    Mar 30 at 12:14










  • $begingroup$
    No, not a priori. Again, what we're really asking is this: if $V$ is a totally decomposable vector bundle on some projective $k$-variety $X$ then does the algebraic $k$-group $Rmapsto mathrmAut(V_R)$ have any non-trivial torsors? I mean, is this group connected? Why can't it be $mathrmPGL_n$? Do you know any structural results about what group this can be?
    $endgroup$
    – Alex Youcis
    Mar 31 at 1:54










  • $begingroup$
    Just to point out, I think that the group $G_V(R):=mathrmAut(V_R)$ contains a central copy of $mathbbG_m$, so it can't be $mathrmPGL_n$.
    $endgroup$
    – Alex Youcis
    Mar 31 at 4:24















6












$begingroup$


Let $k$ be a number field, and let $X$ be a projective $k$-variety. Let $mathcalV$ be a vector bundle on $X$ that is defined over $k$. A vector bundle $mathcalV'$ on $X$ that is defined over $k$ is said to be a twist of $mathcalV$ if there is some finite extension $ell/k$ such that we have an isomorphism $mathcalV_ell simeq mathcalV'_ell$ of bundles upon basechanging to $ell$.



Question: Is it true that all twists of $mathcalV$ are in fact isomorphic to $mathcalV$ over $k$?



What I know: Twists are parameterized by the Galois cohomology group $H^1(operatornameGal(overlinek/k),operatornameAut(calV_overlinek))$, so all we need to do is determine whether this cohomology group is equal to $0$. It's a consequence of Hilbert Theorem 90 that $H^1(operatornameGal(overlinek/k),operatornameGL_n(overlinek)) = 0$, which might be useful.










share|cite|improve this question









$endgroup$







  • 1




    $begingroup$
    And you really need this full statement? I see how to prove the answer in the affirmative if you assume that $X$ is integral and either you're dealing with line bundles or one of your vector bundles is trivial. In the complete general case this, at first glance, seems kind of tough. You're essentially asking about the Galois cohomology of the algebraic group $Rmapsto Aut(mathcalV_R)$. Are there large constraints on what this algebraic group can be?
    $endgroup$
    – Alex Youcis
    Mar 30 at 6:12











  • $begingroup$
    @AlexYoucis I'm happy to assume $X$ is integral, and I think I know how to handle the cases where $calV$ is a line bundle or is trivial. I'm interested primarily in the case where $calV$ splits completely as a direct sum of line bundles, but I'm not sure whether that constrains $operatornameAut(V)$ sufficiently. Does your argument for the line bundle case generalize to the case of the direct sum of line bundles?
    $endgroup$
    – Ashvin Swaminathan
    Mar 30 at 12:14










  • $begingroup$
    No, not a priori. Again, what we're really asking is this: if $V$ is a totally decomposable vector bundle on some projective $k$-variety $X$ then does the algebraic $k$-group $Rmapsto mathrmAut(V_R)$ have any non-trivial torsors? I mean, is this group connected? Why can't it be $mathrmPGL_n$? Do you know any structural results about what group this can be?
    $endgroup$
    – Alex Youcis
    Mar 31 at 1:54










  • $begingroup$
    Just to point out, I think that the group $G_V(R):=mathrmAut(V_R)$ contains a central copy of $mathbbG_m$, so it can't be $mathrmPGL_n$.
    $endgroup$
    – Alex Youcis
    Mar 31 at 4:24













6












6








6


4



$begingroup$


Let $k$ be a number field, and let $X$ be a projective $k$-variety. Let $mathcalV$ be a vector bundle on $X$ that is defined over $k$. A vector bundle $mathcalV'$ on $X$ that is defined over $k$ is said to be a twist of $mathcalV$ if there is some finite extension $ell/k$ such that we have an isomorphism $mathcalV_ell simeq mathcalV'_ell$ of bundles upon basechanging to $ell$.



Question: Is it true that all twists of $mathcalV$ are in fact isomorphic to $mathcalV$ over $k$?



What I know: Twists are parameterized by the Galois cohomology group $H^1(operatornameGal(overlinek/k),operatornameAut(calV_overlinek))$, so all we need to do is determine whether this cohomology group is equal to $0$. It's a consequence of Hilbert Theorem 90 that $H^1(operatornameGal(overlinek/k),operatornameGL_n(overlinek)) = 0$, which might be useful.










share|cite|improve this question









$endgroup$




Let $k$ be a number field, and let $X$ be a projective $k$-variety. Let $mathcalV$ be a vector bundle on $X$ that is defined over $k$. A vector bundle $mathcalV'$ on $X$ that is defined over $k$ is said to be a twist of $mathcalV$ if there is some finite extension $ell/k$ such that we have an isomorphism $mathcalV_ell simeq mathcalV'_ell$ of bundles upon basechanging to $ell$.



Question: Is it true that all twists of $mathcalV$ are in fact isomorphic to $mathcalV$ over $k$?



What I know: Twists are parameterized by the Galois cohomology group $H^1(operatornameGal(overlinek/k),operatornameAut(calV_overlinek))$, so all we need to do is determine whether this cohomology group is equal to $0$. It's a consequence of Hilbert Theorem 90 that $H^1(operatornameGal(overlinek/k),operatornameGL_n(overlinek)) = 0$, which might be useful.







number-theory algebraic-geometry galois-theory vector-bundles galois-cohomology






share|cite|improve this question













share|cite|improve this question











share|cite|improve this question




share|cite|improve this question










asked Mar 30 at 2:16









Ashvin SwaminathanAshvin Swaminathan

1,700520




1,700520







  • 1




    $begingroup$
    And you really need this full statement? I see how to prove the answer in the affirmative if you assume that $X$ is integral and either you're dealing with line bundles or one of your vector bundles is trivial. In the complete general case this, at first glance, seems kind of tough. You're essentially asking about the Galois cohomology of the algebraic group $Rmapsto Aut(mathcalV_R)$. Are there large constraints on what this algebraic group can be?
    $endgroup$
    – Alex Youcis
    Mar 30 at 6:12











  • $begingroup$
    @AlexYoucis I'm happy to assume $X$ is integral, and I think I know how to handle the cases where $calV$ is a line bundle or is trivial. I'm interested primarily in the case where $calV$ splits completely as a direct sum of line bundles, but I'm not sure whether that constrains $operatornameAut(V)$ sufficiently. Does your argument for the line bundle case generalize to the case of the direct sum of line bundles?
    $endgroup$
    – Ashvin Swaminathan
    Mar 30 at 12:14










  • $begingroup$
    No, not a priori. Again, what we're really asking is this: if $V$ is a totally decomposable vector bundle on some projective $k$-variety $X$ then does the algebraic $k$-group $Rmapsto mathrmAut(V_R)$ have any non-trivial torsors? I mean, is this group connected? Why can't it be $mathrmPGL_n$? Do you know any structural results about what group this can be?
    $endgroup$
    – Alex Youcis
    Mar 31 at 1:54










  • $begingroup$
    Just to point out, I think that the group $G_V(R):=mathrmAut(V_R)$ contains a central copy of $mathbbG_m$, so it can't be $mathrmPGL_n$.
    $endgroup$
    – Alex Youcis
    Mar 31 at 4:24












  • 1




    $begingroup$
    And you really need this full statement? I see how to prove the answer in the affirmative if you assume that $X$ is integral and either you're dealing with line bundles or one of your vector bundles is trivial. In the complete general case this, at first glance, seems kind of tough. You're essentially asking about the Galois cohomology of the algebraic group $Rmapsto Aut(mathcalV_R)$. Are there large constraints on what this algebraic group can be?
    $endgroup$
    – Alex Youcis
    Mar 30 at 6:12











  • $begingroup$
    @AlexYoucis I'm happy to assume $X$ is integral, and I think I know how to handle the cases where $calV$ is a line bundle or is trivial. I'm interested primarily in the case where $calV$ splits completely as a direct sum of line bundles, but I'm not sure whether that constrains $operatornameAut(V)$ sufficiently. Does your argument for the line bundle case generalize to the case of the direct sum of line bundles?
    $endgroup$
    – Ashvin Swaminathan
    Mar 30 at 12:14










  • $begingroup$
    No, not a priori. Again, what we're really asking is this: if $V$ is a totally decomposable vector bundle on some projective $k$-variety $X$ then does the algebraic $k$-group $Rmapsto mathrmAut(V_R)$ have any non-trivial torsors? I mean, is this group connected? Why can't it be $mathrmPGL_n$? Do you know any structural results about what group this can be?
    $endgroup$
    – Alex Youcis
    Mar 31 at 1:54










  • $begingroup$
    Just to point out, I think that the group $G_V(R):=mathrmAut(V_R)$ contains a central copy of $mathbbG_m$, so it can't be $mathrmPGL_n$.
    $endgroup$
    – Alex Youcis
    Mar 31 at 4:24







1




1




$begingroup$
And you really need this full statement? I see how to prove the answer in the affirmative if you assume that $X$ is integral and either you're dealing with line bundles or one of your vector bundles is trivial. In the complete general case this, at first glance, seems kind of tough. You're essentially asking about the Galois cohomology of the algebraic group $Rmapsto Aut(mathcalV_R)$. Are there large constraints on what this algebraic group can be?
$endgroup$
– Alex Youcis
Mar 30 at 6:12





$begingroup$
And you really need this full statement? I see how to prove the answer in the affirmative if you assume that $X$ is integral and either you're dealing with line bundles or one of your vector bundles is trivial. In the complete general case this, at first glance, seems kind of tough. You're essentially asking about the Galois cohomology of the algebraic group $Rmapsto Aut(mathcalV_R)$. Are there large constraints on what this algebraic group can be?
$endgroup$
– Alex Youcis
Mar 30 at 6:12













$begingroup$
@AlexYoucis I'm happy to assume $X$ is integral, and I think I know how to handle the cases where $calV$ is a line bundle or is trivial. I'm interested primarily in the case where $calV$ splits completely as a direct sum of line bundles, but I'm not sure whether that constrains $operatornameAut(V)$ sufficiently. Does your argument for the line bundle case generalize to the case of the direct sum of line bundles?
$endgroup$
– Ashvin Swaminathan
Mar 30 at 12:14




$begingroup$
@AlexYoucis I'm happy to assume $X$ is integral, and I think I know how to handle the cases where $calV$ is a line bundle or is trivial. I'm interested primarily in the case where $calV$ splits completely as a direct sum of line bundles, but I'm not sure whether that constrains $operatornameAut(V)$ sufficiently. Does your argument for the line bundle case generalize to the case of the direct sum of line bundles?
$endgroup$
– Ashvin Swaminathan
Mar 30 at 12:14












$begingroup$
No, not a priori. Again, what we're really asking is this: if $V$ is a totally decomposable vector bundle on some projective $k$-variety $X$ then does the algebraic $k$-group $Rmapsto mathrmAut(V_R)$ have any non-trivial torsors? I mean, is this group connected? Why can't it be $mathrmPGL_n$? Do you know any structural results about what group this can be?
$endgroup$
– Alex Youcis
Mar 31 at 1:54




$begingroup$
No, not a priori. Again, what we're really asking is this: if $V$ is a totally decomposable vector bundle on some projective $k$-variety $X$ then does the algebraic $k$-group $Rmapsto mathrmAut(V_R)$ have any non-trivial torsors? I mean, is this group connected? Why can't it be $mathrmPGL_n$? Do you know any structural results about what group this can be?
$endgroup$
– Alex Youcis
Mar 31 at 1:54












$begingroup$
Just to point out, I think that the group $G_V(R):=mathrmAut(V_R)$ contains a central copy of $mathbbG_m$, so it can't be $mathrmPGL_n$.
$endgroup$
– Alex Youcis
Mar 31 at 4:24




$begingroup$
Just to point out, I think that the group $G_V(R):=mathrmAut(V_R)$ contains a central copy of $mathbbG_m$, so it can't be $mathrmPGL_n$.
$endgroup$
– Alex Youcis
Mar 31 at 4:24










1 Answer
1






active

oldest

votes


















5












$begingroup$

$newcommandGLmathrmGL$$newcommandov[1]overline#1$$newcommandhmathcalO$$newcommandAutmathrmAut$$newcommandGalmathrmGal$$newcommandHommathrmHom$$newcommandAmathbbA$
Thanks to a very helpful conversation with my friend P. Achinger here is a solution assuming that $X$ is geometrically integral (he has a proof even when $X$ is just proper and $k$ is infinite, but below is a simplified version in the case when geometrically integral case when $k$ is infinite).



Assume that $k$ is infinite.



Step 1: The assertion is true for line bundles. Indeed, if $L_ovkcong L'_ovk$ then $(Lotimes L^-1)_ovkcong (h_X)_ovk$. But, this then defines a a class of $H^1(Gal(ovk/k),Aut(h_X_ovk))=H^1(Gal(ovk/k),ovk^times)=0$. It follows that $Lcong L'$.



Step 2: Let $V$ and $V'$ be vector bundles on $X$ such that $V_ovkcong V'_ovk$. Note then that $det(V)_ovkcong det(V')_ovk$ so that, by Step 1, we have that $det(V)cong det(V')$. Let us now consider the functor $Rmapsto Hom(V_R,V'_R)$. Note that by flat base change we have that this functor coincides with $Rmapsto Hom(V,V')otimes_k R$ and thus coincides with $A^n_k$ where $n:=dim Hom(V,V')$ (which is finite-dimensional since $X$ is proper). Let $Usubseteq A^n_k$ be defined as the functor
$$U(R)=finHom(V_R,V'_R):ftext isomorphism$$



We claim that this is an open subscheme of $A^n_k$. Indeed, note that we have a functorial map



$$beginalignedA^n_k =&Hom(V,V')toHom(det(V),det(V))=A^1_k\ &:varphimapstodet(varphi)endaligned$$



(Where we've used the fact that $X$ is geometrically integral to identify this last term with $A^1_k$). It's clear then that $U$ is the open subscheme of $A^n_k$ obtained as the preimage of



$$mathbbG_msubseteq A^1_k=Hom(det(V),det(V))$$



Now by assumption we have that $U(ovk)ne varnothing$ so that $U$ is non-empty. This implies, since $k$ is infinite, that $U(k)ne varnothing$ (e.g see this). Thus, $Vcong V'$.



EDIT: Here's a proof that works when $k$ is finite, and thus all cases are covered.



Let us denote by $G_V$ the group scheme over $k$ given by $Rmapsto Aut(V_R)$. Note that, as above, we have that $G_V$ is an open subscheme of the functor $Rmapsto mathrmEnd(V_R)$ which is identified with $A^n_k$ where $n:=dimmathrmEnd(V)$. In particular, since $A^n_k$ is irreducible we deduce that $G_V$ is connected.



Note then that since the twists of $V$ are classified by $H^1_mathrmet(mathrmSpec(k),G_V)$ it suffices to prove that this latter cohomology group is zero. Since $G_V$ is connected this follows from Lang's Theorem.



Remark: It's still a little miraculous that even though the group $G_V:Rmapsto mathrmAut(V_R)$ is mysterious (at least to me) we can prove that $H^1_mathrmet(mathrmSpec(k),G_V)=0$. Does anyone have anything substantive to say about the structure of the group $G_V$? Is it affine? Is it reductive (note that it can't be semisimple because we have a central embedding of $mathbbG_m$ into $G_V$)? If so, is it split? For example, what made the line bundle case so simple is that for any line bundle $L$ we have that $G_LcongmathbbG_m$. Unless I made a mistake, it seems like that it needn't be reductive in general since some examples on $mathbbP^1$ lead me to believe that you can get things like parabolic groups in $GL_n$.






share|cite|improve this answer











$endgroup$













    Your Answer





    StackExchange.ifUsing("editor", function ()
    return StackExchange.using("mathjaxEditing", function ()
    StackExchange.MarkdownEditor.creationCallbacks.add(function (editor, postfix)
    StackExchange.mathjaxEditing.prepareWmdForMathJax(editor, postfix, [["$", "$"], ["\\(","\\)"]]);
    );
    );
    , "mathjax-editing");

    StackExchange.ready(function()
    var channelOptions =
    tags: "".split(" "),
    id: "69"
    ;
    initTagRenderer("".split(" "), "".split(" "), channelOptions);

    StackExchange.using("externalEditor", function()
    // Have to fire editor after snippets, if snippets enabled
    if (StackExchange.settings.snippets.snippetsEnabled)
    StackExchange.using("snippets", function()
    createEditor();
    );

    else
    createEditor();

    );

    function createEditor()
    StackExchange.prepareEditor(
    heartbeatType: 'answer',
    autoActivateHeartbeat: false,
    convertImagesToLinks: true,
    noModals: true,
    showLowRepImageUploadWarning: true,
    reputationToPostImages: 10,
    bindNavPrevention: true,
    postfix: "",
    imageUploader:
    brandingHtml: "Powered by u003ca class="icon-imgur-white" href="https://imgur.com/"u003eu003c/au003e",
    contentPolicyHtml: "User contributions licensed under u003ca href="https://creativecommons.org/licenses/by-sa/3.0/"u003ecc by-sa 3.0 with attribution requiredu003c/au003e u003ca href="https://stackoverflow.com/legal/content-policy"u003e(content policy)u003c/au003e",
    allowUrls: true
    ,
    noCode: true, onDemand: true,
    discardSelector: ".discard-answer"
    ,immediatelyShowMarkdownHelp:true
    );



    );













    draft saved

    draft discarded


















    StackExchange.ready(
    function ()
    StackExchange.openid.initPostLogin('.new-post-login', 'https%3a%2f%2fmath.stackexchange.com%2fquestions%2f3167841%2fnontrivial-twists-of-a-vector-bundle%23new-answer', 'question_page');

    );

    Post as a guest















    Required, but never shown

























    1 Answer
    1






    active

    oldest

    votes








    1 Answer
    1






    active

    oldest

    votes









    active

    oldest

    votes






    active

    oldest

    votes









    5












    $begingroup$

    $newcommandGLmathrmGL$$newcommandov[1]overline#1$$newcommandhmathcalO$$newcommandAutmathrmAut$$newcommandGalmathrmGal$$newcommandHommathrmHom$$newcommandAmathbbA$
    Thanks to a very helpful conversation with my friend P. Achinger here is a solution assuming that $X$ is geometrically integral (he has a proof even when $X$ is just proper and $k$ is infinite, but below is a simplified version in the case when geometrically integral case when $k$ is infinite).



    Assume that $k$ is infinite.



    Step 1: The assertion is true for line bundles. Indeed, if $L_ovkcong L'_ovk$ then $(Lotimes L^-1)_ovkcong (h_X)_ovk$. But, this then defines a a class of $H^1(Gal(ovk/k),Aut(h_X_ovk))=H^1(Gal(ovk/k),ovk^times)=0$. It follows that $Lcong L'$.



    Step 2: Let $V$ and $V'$ be vector bundles on $X$ such that $V_ovkcong V'_ovk$. Note then that $det(V)_ovkcong det(V')_ovk$ so that, by Step 1, we have that $det(V)cong det(V')$. Let us now consider the functor $Rmapsto Hom(V_R,V'_R)$. Note that by flat base change we have that this functor coincides with $Rmapsto Hom(V,V')otimes_k R$ and thus coincides with $A^n_k$ where $n:=dim Hom(V,V')$ (which is finite-dimensional since $X$ is proper). Let $Usubseteq A^n_k$ be defined as the functor
    $$U(R)=finHom(V_R,V'_R):ftext isomorphism$$



    We claim that this is an open subscheme of $A^n_k$. Indeed, note that we have a functorial map



    $$beginalignedA^n_k =&Hom(V,V')toHom(det(V),det(V))=A^1_k\ &:varphimapstodet(varphi)endaligned$$



    (Where we've used the fact that $X$ is geometrically integral to identify this last term with $A^1_k$). It's clear then that $U$ is the open subscheme of $A^n_k$ obtained as the preimage of



    $$mathbbG_msubseteq A^1_k=Hom(det(V),det(V))$$



    Now by assumption we have that $U(ovk)ne varnothing$ so that $U$ is non-empty. This implies, since $k$ is infinite, that $U(k)ne varnothing$ (e.g see this). Thus, $Vcong V'$.



    EDIT: Here's a proof that works when $k$ is finite, and thus all cases are covered.



    Let us denote by $G_V$ the group scheme over $k$ given by $Rmapsto Aut(V_R)$. Note that, as above, we have that $G_V$ is an open subscheme of the functor $Rmapsto mathrmEnd(V_R)$ which is identified with $A^n_k$ where $n:=dimmathrmEnd(V)$. In particular, since $A^n_k$ is irreducible we deduce that $G_V$ is connected.



    Note then that since the twists of $V$ are classified by $H^1_mathrmet(mathrmSpec(k),G_V)$ it suffices to prove that this latter cohomology group is zero. Since $G_V$ is connected this follows from Lang's Theorem.



    Remark: It's still a little miraculous that even though the group $G_V:Rmapsto mathrmAut(V_R)$ is mysterious (at least to me) we can prove that $H^1_mathrmet(mathrmSpec(k),G_V)=0$. Does anyone have anything substantive to say about the structure of the group $G_V$? Is it affine? Is it reductive (note that it can't be semisimple because we have a central embedding of $mathbbG_m$ into $G_V$)? If so, is it split? For example, what made the line bundle case so simple is that for any line bundle $L$ we have that $G_LcongmathbbG_m$. Unless I made a mistake, it seems like that it needn't be reductive in general since some examples on $mathbbP^1$ lead me to believe that you can get things like parabolic groups in $GL_n$.






    share|cite|improve this answer











    $endgroup$

















      5












      $begingroup$

      $newcommandGLmathrmGL$$newcommandov[1]overline#1$$newcommandhmathcalO$$newcommandAutmathrmAut$$newcommandGalmathrmGal$$newcommandHommathrmHom$$newcommandAmathbbA$
      Thanks to a very helpful conversation with my friend P. Achinger here is a solution assuming that $X$ is geometrically integral (he has a proof even when $X$ is just proper and $k$ is infinite, but below is a simplified version in the case when geometrically integral case when $k$ is infinite).



      Assume that $k$ is infinite.



      Step 1: The assertion is true for line bundles. Indeed, if $L_ovkcong L'_ovk$ then $(Lotimes L^-1)_ovkcong (h_X)_ovk$. But, this then defines a a class of $H^1(Gal(ovk/k),Aut(h_X_ovk))=H^1(Gal(ovk/k),ovk^times)=0$. It follows that $Lcong L'$.



      Step 2: Let $V$ and $V'$ be vector bundles on $X$ such that $V_ovkcong V'_ovk$. Note then that $det(V)_ovkcong det(V')_ovk$ so that, by Step 1, we have that $det(V)cong det(V')$. Let us now consider the functor $Rmapsto Hom(V_R,V'_R)$. Note that by flat base change we have that this functor coincides with $Rmapsto Hom(V,V')otimes_k R$ and thus coincides with $A^n_k$ where $n:=dim Hom(V,V')$ (which is finite-dimensional since $X$ is proper). Let $Usubseteq A^n_k$ be defined as the functor
      $$U(R)=finHom(V_R,V'_R):ftext isomorphism$$



      We claim that this is an open subscheme of $A^n_k$. Indeed, note that we have a functorial map



      $$beginalignedA^n_k =&Hom(V,V')toHom(det(V),det(V))=A^1_k\ &:varphimapstodet(varphi)endaligned$$



      (Where we've used the fact that $X$ is geometrically integral to identify this last term with $A^1_k$). It's clear then that $U$ is the open subscheme of $A^n_k$ obtained as the preimage of



      $$mathbbG_msubseteq A^1_k=Hom(det(V),det(V))$$



      Now by assumption we have that $U(ovk)ne varnothing$ so that $U$ is non-empty. This implies, since $k$ is infinite, that $U(k)ne varnothing$ (e.g see this). Thus, $Vcong V'$.



      EDIT: Here's a proof that works when $k$ is finite, and thus all cases are covered.



      Let us denote by $G_V$ the group scheme over $k$ given by $Rmapsto Aut(V_R)$. Note that, as above, we have that $G_V$ is an open subscheme of the functor $Rmapsto mathrmEnd(V_R)$ which is identified with $A^n_k$ where $n:=dimmathrmEnd(V)$. In particular, since $A^n_k$ is irreducible we deduce that $G_V$ is connected.



      Note then that since the twists of $V$ are classified by $H^1_mathrmet(mathrmSpec(k),G_V)$ it suffices to prove that this latter cohomology group is zero. Since $G_V$ is connected this follows from Lang's Theorem.



      Remark: It's still a little miraculous that even though the group $G_V:Rmapsto mathrmAut(V_R)$ is mysterious (at least to me) we can prove that $H^1_mathrmet(mathrmSpec(k),G_V)=0$. Does anyone have anything substantive to say about the structure of the group $G_V$? Is it affine? Is it reductive (note that it can't be semisimple because we have a central embedding of $mathbbG_m$ into $G_V$)? If so, is it split? For example, what made the line bundle case so simple is that for any line bundle $L$ we have that $G_LcongmathbbG_m$. Unless I made a mistake, it seems like that it needn't be reductive in general since some examples on $mathbbP^1$ lead me to believe that you can get things like parabolic groups in $GL_n$.






      share|cite|improve this answer











      $endgroup$















        5












        5








        5





        $begingroup$

        $newcommandGLmathrmGL$$newcommandov[1]overline#1$$newcommandhmathcalO$$newcommandAutmathrmAut$$newcommandGalmathrmGal$$newcommandHommathrmHom$$newcommandAmathbbA$
        Thanks to a very helpful conversation with my friend P. Achinger here is a solution assuming that $X$ is geometrically integral (he has a proof even when $X$ is just proper and $k$ is infinite, but below is a simplified version in the case when geometrically integral case when $k$ is infinite).



        Assume that $k$ is infinite.



        Step 1: The assertion is true for line bundles. Indeed, if $L_ovkcong L'_ovk$ then $(Lotimes L^-1)_ovkcong (h_X)_ovk$. But, this then defines a a class of $H^1(Gal(ovk/k),Aut(h_X_ovk))=H^1(Gal(ovk/k),ovk^times)=0$. It follows that $Lcong L'$.



        Step 2: Let $V$ and $V'$ be vector bundles on $X$ such that $V_ovkcong V'_ovk$. Note then that $det(V)_ovkcong det(V')_ovk$ so that, by Step 1, we have that $det(V)cong det(V')$. Let us now consider the functor $Rmapsto Hom(V_R,V'_R)$. Note that by flat base change we have that this functor coincides with $Rmapsto Hom(V,V')otimes_k R$ and thus coincides with $A^n_k$ where $n:=dim Hom(V,V')$ (which is finite-dimensional since $X$ is proper). Let $Usubseteq A^n_k$ be defined as the functor
        $$U(R)=finHom(V_R,V'_R):ftext isomorphism$$



        We claim that this is an open subscheme of $A^n_k$. Indeed, note that we have a functorial map



        $$beginalignedA^n_k =&Hom(V,V')toHom(det(V),det(V))=A^1_k\ &:varphimapstodet(varphi)endaligned$$



        (Where we've used the fact that $X$ is geometrically integral to identify this last term with $A^1_k$). It's clear then that $U$ is the open subscheme of $A^n_k$ obtained as the preimage of



        $$mathbbG_msubseteq A^1_k=Hom(det(V),det(V))$$



        Now by assumption we have that $U(ovk)ne varnothing$ so that $U$ is non-empty. This implies, since $k$ is infinite, that $U(k)ne varnothing$ (e.g see this). Thus, $Vcong V'$.



        EDIT: Here's a proof that works when $k$ is finite, and thus all cases are covered.



        Let us denote by $G_V$ the group scheme over $k$ given by $Rmapsto Aut(V_R)$. Note that, as above, we have that $G_V$ is an open subscheme of the functor $Rmapsto mathrmEnd(V_R)$ which is identified with $A^n_k$ where $n:=dimmathrmEnd(V)$. In particular, since $A^n_k$ is irreducible we deduce that $G_V$ is connected.



        Note then that since the twists of $V$ are classified by $H^1_mathrmet(mathrmSpec(k),G_V)$ it suffices to prove that this latter cohomology group is zero. Since $G_V$ is connected this follows from Lang's Theorem.



        Remark: It's still a little miraculous that even though the group $G_V:Rmapsto mathrmAut(V_R)$ is mysterious (at least to me) we can prove that $H^1_mathrmet(mathrmSpec(k),G_V)=0$. Does anyone have anything substantive to say about the structure of the group $G_V$? Is it affine? Is it reductive (note that it can't be semisimple because we have a central embedding of $mathbbG_m$ into $G_V$)? If so, is it split? For example, what made the line bundle case so simple is that for any line bundle $L$ we have that $G_LcongmathbbG_m$. Unless I made a mistake, it seems like that it needn't be reductive in general since some examples on $mathbbP^1$ lead me to believe that you can get things like parabolic groups in $GL_n$.






        share|cite|improve this answer











        $endgroup$



        $newcommandGLmathrmGL$$newcommandov[1]overline#1$$newcommandhmathcalO$$newcommandAutmathrmAut$$newcommandGalmathrmGal$$newcommandHommathrmHom$$newcommandAmathbbA$
        Thanks to a very helpful conversation with my friend P. Achinger here is a solution assuming that $X$ is geometrically integral (he has a proof even when $X$ is just proper and $k$ is infinite, but below is a simplified version in the case when geometrically integral case when $k$ is infinite).



        Assume that $k$ is infinite.



        Step 1: The assertion is true for line bundles. Indeed, if $L_ovkcong L'_ovk$ then $(Lotimes L^-1)_ovkcong (h_X)_ovk$. But, this then defines a a class of $H^1(Gal(ovk/k),Aut(h_X_ovk))=H^1(Gal(ovk/k),ovk^times)=0$. It follows that $Lcong L'$.



        Step 2: Let $V$ and $V'$ be vector bundles on $X$ such that $V_ovkcong V'_ovk$. Note then that $det(V)_ovkcong det(V')_ovk$ so that, by Step 1, we have that $det(V)cong det(V')$. Let us now consider the functor $Rmapsto Hom(V_R,V'_R)$. Note that by flat base change we have that this functor coincides with $Rmapsto Hom(V,V')otimes_k R$ and thus coincides with $A^n_k$ where $n:=dim Hom(V,V')$ (which is finite-dimensional since $X$ is proper). Let $Usubseteq A^n_k$ be defined as the functor
        $$U(R)=finHom(V_R,V'_R):ftext isomorphism$$



        We claim that this is an open subscheme of $A^n_k$. Indeed, note that we have a functorial map



        $$beginalignedA^n_k =&Hom(V,V')toHom(det(V),det(V))=A^1_k\ &:varphimapstodet(varphi)endaligned$$



        (Where we've used the fact that $X$ is geometrically integral to identify this last term with $A^1_k$). It's clear then that $U$ is the open subscheme of $A^n_k$ obtained as the preimage of



        $$mathbbG_msubseteq A^1_k=Hom(det(V),det(V))$$



        Now by assumption we have that $U(ovk)ne varnothing$ so that $U$ is non-empty. This implies, since $k$ is infinite, that $U(k)ne varnothing$ (e.g see this). Thus, $Vcong V'$.



        EDIT: Here's a proof that works when $k$ is finite, and thus all cases are covered.



        Let us denote by $G_V$ the group scheme over $k$ given by $Rmapsto Aut(V_R)$. Note that, as above, we have that $G_V$ is an open subscheme of the functor $Rmapsto mathrmEnd(V_R)$ which is identified with $A^n_k$ where $n:=dimmathrmEnd(V)$. In particular, since $A^n_k$ is irreducible we deduce that $G_V$ is connected.



        Note then that since the twists of $V$ are classified by $H^1_mathrmet(mathrmSpec(k),G_V)$ it suffices to prove that this latter cohomology group is zero. Since $G_V$ is connected this follows from Lang's Theorem.



        Remark: It's still a little miraculous that even though the group $G_V:Rmapsto mathrmAut(V_R)$ is mysterious (at least to me) we can prove that $H^1_mathrmet(mathrmSpec(k),G_V)=0$. Does anyone have anything substantive to say about the structure of the group $G_V$? Is it affine? Is it reductive (note that it can't be semisimple because we have a central embedding of $mathbbG_m$ into $G_V$)? If so, is it split? For example, what made the line bundle case so simple is that for any line bundle $L$ we have that $G_LcongmathbbG_m$. Unless I made a mistake, it seems like that it needn't be reductive in general since some examples on $mathbbP^1$ lead me to believe that you can get things like parabolic groups in $GL_n$.







        share|cite|improve this answer














        share|cite|improve this answer



        share|cite|improve this answer








        edited Apr 3 at 8:06

























        answered Mar 31 at 5:10









        Alex YoucisAlex Youcis

        36.1k775115




        36.1k775115



























            draft saved

            draft discarded
















































            Thanks for contributing an answer to Mathematics Stack Exchange!


            • Please be sure to answer the question. Provide details and share your research!

            But avoid


            • Asking for help, clarification, or responding to other answers.

            • Making statements based on opinion; back them up with references or personal experience.

            Use MathJax to format equations. MathJax reference.


            To learn more, see our tips on writing great answers.




            draft saved


            draft discarded














            StackExchange.ready(
            function ()
            StackExchange.openid.initPostLogin('.new-post-login', 'https%3a%2f%2fmath.stackexchange.com%2fquestions%2f3167841%2fnontrivial-twists-of-a-vector-bundle%23new-answer', 'question_page');

            );

            Post as a guest















            Required, but never shown





















































            Required, but never shown














            Required, but never shown












            Required, but never shown







            Required, but never shown

































            Required, but never shown














            Required, but never shown












            Required, but never shown







            Required, but never shown







            Popular posts from this blog

            Triangular numbers and gcdProving sum of a set is $0 pmod n$ if $n$ is odd, or $fracn2 pmod n$ if $n$ is even?Is greatest common divisor of two numbers really their smallest linear combination?GCD, LCM RelationshipProve a set of nonnegative integers with greatest common divisor 1 and closed under addition has all but finite many nonnegative integers.all pairs of a and b in an equation containing gcdTriangular Numbers Modulo $k$ - Hit All Values?Understanding the Existence and Uniqueness of the GCDGCD and LCM with logical symbolsThe greatest common divisor of two positive integers less than 100 is equal to 3. Their least common multiple is twelve times one of the integers.Suppose that for all integers $x$, $x|a$ and $x|b$ if and only if $x|c$. Then $c = gcd(a,b)$Which is the gcd of 2 numbers which are multiplied and the result is 600000?

            Ingelân Ynhâld Etymology | Geografy | Skiednis | Polityk en bestjoer | Ekonomy | Demografy | Kultuer | Klimaat | Sjoch ek | Keppelings om utens | Boarnen, noaten en referinsjes Navigaasjemenuwww.gov.ukOffisjele webside fan it regear fan it Feriene KeninkrykOffisjele webside fan it Britske FerkearsburoNederlânsktalige ynformaasje fan it Britske FerkearsburoOffisjele webside fan English Heritage, de organisaasje dy't him ynset foar it behâld fan it Ingelske kultuergoedYnwennertallen fan alle Britske stêden út 'e folkstelling fan 2011Notes en References, op dizze sideEngland

            Հադիս Բովանդակություն Անվանում և նշանակություն | Դասակարգում | Աղբյուրներ | Նավարկման ցանկ